LSAT and Law School Admissions Forum

Get expert LSAT preparation and law school admissions advice from PowerScore Test Preparation.

 Administrator
PowerScore Staff
  • PowerScore Staff
  • Posts: 8919
  • Joined: Feb 02, 2011
|
#33369
Complete Question Explanation

Strengthen. The correct answer choice is (B)

This example deals with an ancient abandoned Roman well and a few things that were found inside: kitchen items made from a copper alloy, and below that, several coins that included some dating back to 375 a.d. Based on the age of the coins and the fact that the kitchen implements were found above them, the author concludes that the earliest the implements could have been dropped into the well would have been 375 a.d.

The author’s reasoning is based on the idea that the kitchen items could not have predated the coins because they were found above the coins, closer to the surface; by this reasoning, for example, the kitchen implements could not have been dropped into the well in 374 a.d., because the coins that were found below had not yet been created.

The stimulus is followed by a Strengthen question, so the correct answer choice will bolster the author’s assertion that the dishes must have been dropped into the well after the coins, or it will help to rule out the possibility that the dishes were dropped in beforehand.

Answer choice (A): The amount of time that coins spent in circulation is irrelevant, since the author’s conclusion is that the implements above the coins were dropped into the well no earlier than 375 a.d. (thus, even if the coins dated 375 a.d. had been in circulation until 395 a.d., for example, that would have no effect on the strength of the argument).

Answer choice (B): This is the correct answer choice, because it helps to rule out an alternate possibility. If the coins could not have been dropped into the Roman well after the kitchen items, that strengthens the author’s argument, which is based on the notion that the coins were dropped in before the kitchen items.

Answer choice (C): A comparison of the value of the respective items is irrelevant to the author’s argument, which is based solely on the items’ placement in the well. Since this choice does not strengthen the author’s argument, it cannot be the right answer to this Strengthen question.

Answer choice (D): The author does not discuss why the items remained in the well, so this answer, which provides that the items were probably thrown into the well during an evacuation, and would have been retrieved later if the people had returned, does not strengthen the argument in any way.

Answer choice (E): The fact that there were even older items beneath the coins does not strengthen the author’s conclusion, which is that the kitchen items could not have been placed into the well earlier than 375 a.d.
 fg6118
  • Posts: 26
  • Joined: Apr 18, 2016
|
#23989
In preptest 69 LR I q. 13, I chose A instead of B. I see why B strengthens the conclusion, "the implements, therefore, were dropped into the well no earlier than 375 A.D." I don't see why A doesn't strengthen it and why B is better than A. I comfortably eliminated c,d and e. I identified an assumption of the author to be that the implement layer above the coin layer is younger than the coin layer. If coins used in the Roman Empire remain in circulation, those coins that were dated to 375 could've been put into the well in a year later than 375. This makes it more likely that the layer above those coins (the implements) are from a date later than 375.

I recognize that A leaves us with some uncertainty in that it says "often remained," leaving room for some coins to not continue to be circulated and the coins in question could be of the kind that didn't remain in circulation. However I see similar doubt in answer B since those coins could've been the kind that remained in circulation, making them older than 375. Those coins then could've been put into the well, purposefully placed under the implements, which doesn't give us any clues about the age of the implements.

What am I not seeing/thinking through with this question? Thanks for the help!
 Robert Carroll
PowerScore Staff
  • PowerScore Staff
  • Posts: 1787
  • Joined: Dec 06, 2013
|
#24100
Hi fg,

The issue with answer choice (A) is that, even granting your point (that, because they could stay in circulation for a while, the coins dated 375 AD might have been dropped into the well later than 375 AD), this does not make it any more likely that the implement layer was younger than the coin layer. Given the facts as they stand in the stimulus, there were some coins that dated to 375 AD. Whether or not coins often remained in circulation for many decades, it is possible that those coins were placed into the well any time from 375 AD to the present. We don't have any reason to think that some coins' being in circulation for a given time will make the coins in the well more likely to have been placed there later. Thus, answer choice (A) and the stimulus are entirely consistent with the coins' having been placed in the well any time from 375 AD to the present, and the information in answer choice (A) does not, by itself, affect our prediction of when the coins were placed in the well.

If the implements were placed in the well after the coins, and at least some coins date back to 375 AD, then there is no chance the implements were placed in the well before 375 AD. That the implements were on top of the coins is not proof of this - it is perhaps possible that the implements were placed in first and that the coins slipped to a lower layer for some reason. Thus, the argument has (at least) this weakness - something may have caused the coins to shift position from where they originally were. Answer choice (B) eliminates that weakness. Thus, it strengthens the argument. Because answer choice (A) does not strengthen it because it does not give any basis for considering the conclusion more likely to be true (no information is given that allows us to tell the coins were put in later than 375 AD), answer choice (A) is incorrect. Answer choice (B) at least defuses one possible weakness, which strengthens the argument.

The doubt that remains about someone purposely putting the coins underneath the implements is another possible weakness with the argument, but neither answer choice that we discussed deals with this weakness. Answer choice (A) doesn't dispel it at all. Thus, if we're choosing between the two answer choices on the basis of that, we have no additional reason to choose answer choice (A). Answer choice (B) is still correct because, while it does not get rid of every weakness, it doesn't need to do that - this is a Strengthen question, not a Justify question.

Robert Carroll
 fg6118
  • Posts: 26
  • Joined: Apr 18, 2016
|
#24102
Thanks for the thorough explanation, Robert. It helped me understand B's strength over A.
 Robert Carroll
PowerScore Staff
  • PowerScore Staff
  • Posts: 1787
  • Joined: Dec 06, 2013
|
#24103
Sure thing!

Robert Carroll

Get the most out of your LSAT Prep Plus subscription.

Analyze and track your performance with our Testing and Analytics Package.